Infinite series problem












7












$begingroup$



The sum of $$frac{2}{4-1}+frac{2^2}{4^2-1}+frac{2^4}{4^4-1}+cdots cdots $$




Try: write it as $$S = sum^{infty}_{r=0}frac{2^{2^{r}}}{2^{2^{r+1}}-1}=sum^{infty}_{r=0}frac{2^{2^r}-1+1}{(2^{2^r}-1)(2^{2^r}+1)}$$



d not know how to solve from here, could some help me



to solve it, Thanks










share|cite|improve this question









$endgroup$








  • 1




    $begingroup$
    $$dfrac a{a^2-1}-dfrac a{a^2+1}=dfrac 2{a^4-1}$$
    $endgroup$
    – lab bhattacharjee
    Jan 24 at 13:07










  • $begingroup$
    @labbhattacharjee I think he had the right side and wrote the left one...
    $endgroup$
    – DonAntonio
    Jan 24 at 13:09
















7












$begingroup$



The sum of $$frac{2}{4-1}+frac{2^2}{4^2-1}+frac{2^4}{4^4-1}+cdots cdots $$




Try: write it as $$S = sum^{infty}_{r=0}frac{2^{2^{r}}}{2^{2^{r+1}}-1}=sum^{infty}_{r=0}frac{2^{2^r}-1+1}{(2^{2^r}-1)(2^{2^r}+1)}$$



d not know how to solve from here, could some help me



to solve it, Thanks










share|cite|improve this question









$endgroup$








  • 1




    $begingroup$
    $$dfrac a{a^2-1}-dfrac a{a^2+1}=dfrac 2{a^4-1}$$
    $endgroup$
    – lab bhattacharjee
    Jan 24 at 13:07










  • $begingroup$
    @labbhattacharjee I think he had the right side and wrote the left one...
    $endgroup$
    – DonAntonio
    Jan 24 at 13:09














7












7








7


1



$begingroup$



The sum of $$frac{2}{4-1}+frac{2^2}{4^2-1}+frac{2^4}{4^4-1}+cdots cdots $$




Try: write it as $$S = sum^{infty}_{r=0}frac{2^{2^{r}}}{2^{2^{r+1}}-1}=sum^{infty}_{r=0}frac{2^{2^r}-1+1}{(2^{2^r}-1)(2^{2^r}+1)}$$



d not know how to solve from here, could some help me



to solve it, Thanks










share|cite|improve this question









$endgroup$





The sum of $$frac{2}{4-1}+frac{2^2}{4^2-1}+frac{2^4}{4^4-1}+cdots cdots $$




Try: write it as $$S = sum^{infty}_{r=0}frac{2^{2^{r}}}{2^{2^{r+1}}-1}=sum^{infty}_{r=0}frac{2^{2^r}-1+1}{(2^{2^r}-1)(2^{2^r}+1)}$$



d not know how to solve from here, could some help me



to solve it, Thanks







sequences-and-series






share|cite|improve this question













share|cite|improve this question











share|cite|improve this question




share|cite|improve this question










asked Jan 24 at 13:00









DXTDXT

6,0612732




6,0612732








  • 1




    $begingroup$
    $$dfrac a{a^2-1}-dfrac a{a^2+1}=dfrac 2{a^4-1}$$
    $endgroup$
    – lab bhattacharjee
    Jan 24 at 13:07










  • $begingroup$
    @labbhattacharjee I think he had the right side and wrote the left one...
    $endgroup$
    – DonAntonio
    Jan 24 at 13:09














  • 1




    $begingroup$
    $$dfrac a{a^2-1}-dfrac a{a^2+1}=dfrac 2{a^4-1}$$
    $endgroup$
    – lab bhattacharjee
    Jan 24 at 13:07










  • $begingroup$
    @labbhattacharjee I think he had the right side and wrote the left one...
    $endgroup$
    – DonAntonio
    Jan 24 at 13:09








1




1




$begingroup$
$$dfrac a{a^2-1}-dfrac a{a^2+1}=dfrac 2{a^4-1}$$
$endgroup$
– lab bhattacharjee
Jan 24 at 13:07




$begingroup$
$$dfrac a{a^2-1}-dfrac a{a^2+1}=dfrac 2{a^4-1}$$
$endgroup$
– lab bhattacharjee
Jan 24 at 13:07












$begingroup$
@labbhattacharjee I think he had the right side and wrote the left one...
$endgroup$
– DonAntonio
Jan 24 at 13:09




$begingroup$
@labbhattacharjee I think he had the right side and wrote the left one...
$endgroup$
– DonAntonio
Jan 24 at 13:09










3 Answers
3






active

oldest

votes


















3












$begingroup$

You may continue as:



$$frac{2^n}{(2^n-1)(2^n+1)} =frac{(2^n+1)-1}{(2^n-1)(2^n+1)} =frac 1{2^n-1} - frac 1{(2^n-1)(2^n+1)}$$
Where $n = 2^r$. Now write the sum as:



$$left(frac 11 - frac 13right) + left(frac 13 - frac 1{15}right) + ... + left(frac 1{2^n-1} - frac 1{(2^n-1)(2^n+1)}right)$$



After cancelling the terms you are left with:



$$1 - frac 1{(2^n-1)(2^n+1)}$$



When $n$ tends to infinity, the expression becomes $1$.






share|cite|improve this answer











$endgroup$









  • 1




    $begingroup$
    Welcome to Math.SE. Note that mathematical notation can be properly typeset in posts, which makes them easier to read.
    $endgroup$
    – hardmath
    Jan 24 at 16:40






  • 1




    $begingroup$
    @PaulSinclair In the solution , $n=2^j$ , so the pattern does indeed follow , as $$frac{1}{2^2^{j+1}-1}=frac{1}{(2^2^j+1)(2^2^j-1)}$$ . While the solution should have mentioned this , I hope this clears it up .
    $endgroup$
    – Sinπ
    Jan 24 at 19:05












  • $begingroup$
    @Rahuboy For future reference you can't do double exponentials without braces, e.g. $2^{2^{j+1}}$ is 2^{2^{j+1}}, not 2^2^{j+1}. That's why your comment malfunctioned.
    $endgroup$
    – J.G.
    Jan 24 at 22:29










  • $begingroup$
    @J.G. Oh I see.... thanks for the help !
    $endgroup$
    – Sinπ
    Jan 25 at 0:02










  • $begingroup$
    That is embarrassing. My apologies for failing to realize this.
    $endgroup$
    – Paul Sinclair
    Jan 25 at 0:14



















7












$begingroup$

You may prove by induction that $$sum_{r=0}^nfrac{2^{2^r}}{2^{2^{r+1}}-1}=1-frac{1}{2^{2^{n+1}}-1}.$$(If yoiu didn't spot this conjecture at first, you will after calculating the first few partial sums.) Indeed the claim is correct if $n=0$, and if it holds for $n=k$ then $$sum_{r=0}^{k+1}frac{2^{2^r}}{2^{2^{r+1}}-1}=1-frac{1}{2^{2^{k+1}}-1}+frac{2^{2^{k+1}}}{2^{2^{k+2}}-1}=1-frac{1}{2^{2^{k+2}}-1}$$as required, where the final calculation is the $a=2^{2^{k+1}}$ special case of $$1-frac{1}{a-1}+frac{a}{a^2-1}=1-frac{1}{a^2-1}.$$You may wish to rewrite this argument as the computation of a telescoping series.






share|cite|improve this answer









$endgroup$





















    2












    $begingroup$

    Note first that
    $$
    frac{2^n}{2^{2n}-1}=2^{-n}frac{1}{1-2^{-2n}}=sum_{k:text{odd},kinBbb N} 2^{-nk}.
    $$
    If we sum over $n=2^j$, $jge 0$, we have
    $$
    S=sum_{j=0}^infty frac{2^{2^j}}{2^{2^{j+1}}-1}=sum_{j=0}^inftysum_{k:text{odd},kinBbb N} 2^{-2^jcdot k}=sum_{l=1}^infty 2^{-l}=1
    $$
    since every $lge 1$ has a unique representation $l=2^jcdot k$ for some $jge 0$ and odd $kin Bbb N$.






    share|cite|improve this answer











    $endgroup$













      Your Answer





      StackExchange.ifUsing("editor", function () {
      return StackExchange.using("mathjaxEditing", function () {
      StackExchange.MarkdownEditor.creationCallbacks.add(function (editor, postfix) {
      StackExchange.mathjaxEditing.prepareWmdForMathJax(editor, postfix, [["$", "$"], ["\\(","\\)"]]);
      });
      });
      }, "mathjax-editing");

      StackExchange.ready(function() {
      var channelOptions = {
      tags: "".split(" "),
      id: "69"
      };
      initTagRenderer("".split(" "), "".split(" "), channelOptions);

      StackExchange.using("externalEditor", function() {
      // Have to fire editor after snippets, if snippets enabled
      if (StackExchange.settings.snippets.snippetsEnabled) {
      StackExchange.using("snippets", function() {
      createEditor();
      });
      }
      else {
      createEditor();
      }
      });

      function createEditor() {
      StackExchange.prepareEditor({
      heartbeatType: 'answer',
      autoActivateHeartbeat: false,
      convertImagesToLinks: true,
      noModals: true,
      showLowRepImageUploadWarning: true,
      reputationToPostImages: 10,
      bindNavPrevention: true,
      postfix: "",
      imageUploader: {
      brandingHtml: "Powered by u003ca class="icon-imgur-white" href="https://imgur.com/"u003eu003c/au003e",
      contentPolicyHtml: "User contributions licensed under u003ca href="https://creativecommons.org/licenses/by-sa/3.0/"u003ecc by-sa 3.0 with attribution requiredu003c/au003e u003ca href="https://stackoverflow.com/legal/content-policy"u003e(content policy)u003c/au003e",
      allowUrls: true
      },
      noCode: true, onDemand: true,
      discardSelector: ".discard-answer"
      ,immediatelyShowMarkdownHelp:true
      });


      }
      });














      draft saved

      draft discarded


















      StackExchange.ready(
      function () {
      StackExchange.openid.initPostLogin('.new-post-login', 'https%3a%2f%2fmath.stackexchange.com%2fquestions%2f3085853%2finfinite-series-problem%23new-answer', 'question_page');
      }
      );

      Post as a guest















      Required, but never shown

























      3 Answers
      3






      active

      oldest

      votes








      3 Answers
      3






      active

      oldest

      votes









      active

      oldest

      votes






      active

      oldest

      votes









      3












      $begingroup$

      You may continue as:



      $$frac{2^n}{(2^n-1)(2^n+1)} =frac{(2^n+1)-1}{(2^n-1)(2^n+1)} =frac 1{2^n-1} - frac 1{(2^n-1)(2^n+1)}$$
      Where $n = 2^r$. Now write the sum as:



      $$left(frac 11 - frac 13right) + left(frac 13 - frac 1{15}right) + ... + left(frac 1{2^n-1} - frac 1{(2^n-1)(2^n+1)}right)$$



      After cancelling the terms you are left with:



      $$1 - frac 1{(2^n-1)(2^n+1)}$$



      When $n$ tends to infinity, the expression becomes $1$.






      share|cite|improve this answer











      $endgroup$









      • 1




        $begingroup$
        Welcome to Math.SE. Note that mathematical notation can be properly typeset in posts, which makes them easier to read.
        $endgroup$
        – hardmath
        Jan 24 at 16:40






      • 1




        $begingroup$
        @PaulSinclair In the solution , $n=2^j$ , so the pattern does indeed follow , as $$frac{1}{2^2^{j+1}-1}=frac{1}{(2^2^j+1)(2^2^j-1)}$$ . While the solution should have mentioned this , I hope this clears it up .
        $endgroup$
        – Sinπ
        Jan 24 at 19:05












      • $begingroup$
        @Rahuboy For future reference you can't do double exponentials without braces, e.g. $2^{2^{j+1}}$ is 2^{2^{j+1}}, not 2^2^{j+1}. That's why your comment malfunctioned.
        $endgroup$
        – J.G.
        Jan 24 at 22:29










      • $begingroup$
        @J.G. Oh I see.... thanks for the help !
        $endgroup$
        – Sinπ
        Jan 25 at 0:02










      • $begingroup$
        That is embarrassing. My apologies for failing to realize this.
        $endgroup$
        – Paul Sinclair
        Jan 25 at 0:14
















      3












      $begingroup$

      You may continue as:



      $$frac{2^n}{(2^n-1)(2^n+1)} =frac{(2^n+1)-1}{(2^n-1)(2^n+1)} =frac 1{2^n-1} - frac 1{(2^n-1)(2^n+1)}$$
      Where $n = 2^r$. Now write the sum as:



      $$left(frac 11 - frac 13right) + left(frac 13 - frac 1{15}right) + ... + left(frac 1{2^n-1} - frac 1{(2^n-1)(2^n+1)}right)$$



      After cancelling the terms you are left with:



      $$1 - frac 1{(2^n-1)(2^n+1)}$$



      When $n$ tends to infinity, the expression becomes $1$.






      share|cite|improve this answer











      $endgroup$









      • 1




        $begingroup$
        Welcome to Math.SE. Note that mathematical notation can be properly typeset in posts, which makes them easier to read.
        $endgroup$
        – hardmath
        Jan 24 at 16:40






      • 1




        $begingroup$
        @PaulSinclair In the solution , $n=2^j$ , so the pattern does indeed follow , as $$frac{1}{2^2^{j+1}-1}=frac{1}{(2^2^j+1)(2^2^j-1)}$$ . While the solution should have mentioned this , I hope this clears it up .
        $endgroup$
        – Sinπ
        Jan 24 at 19:05












      • $begingroup$
        @Rahuboy For future reference you can't do double exponentials without braces, e.g. $2^{2^{j+1}}$ is 2^{2^{j+1}}, not 2^2^{j+1}. That's why your comment malfunctioned.
        $endgroup$
        – J.G.
        Jan 24 at 22:29










      • $begingroup$
        @J.G. Oh I see.... thanks for the help !
        $endgroup$
        – Sinπ
        Jan 25 at 0:02










      • $begingroup$
        That is embarrassing. My apologies for failing to realize this.
        $endgroup$
        – Paul Sinclair
        Jan 25 at 0:14














      3












      3








      3





      $begingroup$

      You may continue as:



      $$frac{2^n}{(2^n-1)(2^n+1)} =frac{(2^n+1)-1}{(2^n-1)(2^n+1)} =frac 1{2^n-1} - frac 1{(2^n-1)(2^n+1)}$$
      Where $n = 2^r$. Now write the sum as:



      $$left(frac 11 - frac 13right) + left(frac 13 - frac 1{15}right) + ... + left(frac 1{2^n-1} - frac 1{(2^n-1)(2^n+1)}right)$$



      After cancelling the terms you are left with:



      $$1 - frac 1{(2^n-1)(2^n+1)}$$



      When $n$ tends to infinity, the expression becomes $1$.






      share|cite|improve this answer











      $endgroup$



      You may continue as:



      $$frac{2^n}{(2^n-1)(2^n+1)} =frac{(2^n+1)-1}{(2^n-1)(2^n+1)} =frac 1{2^n-1} - frac 1{(2^n-1)(2^n+1)}$$
      Where $n = 2^r$. Now write the sum as:



      $$left(frac 11 - frac 13right) + left(frac 13 - frac 1{15}right) + ... + left(frac 1{2^n-1} - frac 1{(2^n-1)(2^n+1)}right)$$



      After cancelling the terms you are left with:



      $$1 - frac 1{(2^n-1)(2^n+1)}$$



      When $n$ tends to infinity, the expression becomes $1$.







      share|cite|improve this answer














      share|cite|improve this answer



      share|cite|improve this answer








      edited Jan 25 at 0:21









      Paul Sinclair

      20.5k21543




      20.5k21543










      answered Jan 24 at 16:02









      AspiringEngineerAspiringEngineer

      514




      514








      • 1




        $begingroup$
        Welcome to Math.SE. Note that mathematical notation can be properly typeset in posts, which makes them easier to read.
        $endgroup$
        – hardmath
        Jan 24 at 16:40






      • 1




        $begingroup$
        @PaulSinclair In the solution , $n=2^j$ , so the pattern does indeed follow , as $$frac{1}{2^2^{j+1}-1}=frac{1}{(2^2^j+1)(2^2^j-1)}$$ . While the solution should have mentioned this , I hope this clears it up .
        $endgroup$
        – Sinπ
        Jan 24 at 19:05












      • $begingroup$
        @Rahuboy For future reference you can't do double exponentials without braces, e.g. $2^{2^{j+1}}$ is 2^{2^{j+1}}, not 2^2^{j+1}. That's why your comment malfunctioned.
        $endgroup$
        – J.G.
        Jan 24 at 22:29










      • $begingroup$
        @J.G. Oh I see.... thanks for the help !
        $endgroup$
        – Sinπ
        Jan 25 at 0:02










      • $begingroup$
        That is embarrassing. My apologies for failing to realize this.
        $endgroup$
        – Paul Sinclair
        Jan 25 at 0:14














      • 1




        $begingroup$
        Welcome to Math.SE. Note that mathematical notation can be properly typeset in posts, which makes them easier to read.
        $endgroup$
        – hardmath
        Jan 24 at 16:40






      • 1




        $begingroup$
        @PaulSinclair In the solution , $n=2^j$ , so the pattern does indeed follow , as $$frac{1}{2^2^{j+1}-1}=frac{1}{(2^2^j+1)(2^2^j-1)}$$ . While the solution should have mentioned this , I hope this clears it up .
        $endgroup$
        – Sinπ
        Jan 24 at 19:05












      • $begingroup$
        @Rahuboy For future reference you can't do double exponentials without braces, e.g. $2^{2^{j+1}}$ is 2^{2^{j+1}}, not 2^2^{j+1}. That's why your comment malfunctioned.
        $endgroup$
        – J.G.
        Jan 24 at 22:29










      • $begingroup$
        @J.G. Oh I see.... thanks for the help !
        $endgroup$
        – Sinπ
        Jan 25 at 0:02










      • $begingroup$
        That is embarrassing. My apologies for failing to realize this.
        $endgroup$
        – Paul Sinclair
        Jan 25 at 0:14








      1




      1




      $begingroup$
      Welcome to Math.SE. Note that mathematical notation can be properly typeset in posts, which makes them easier to read.
      $endgroup$
      – hardmath
      Jan 24 at 16:40




      $begingroup$
      Welcome to Math.SE. Note that mathematical notation can be properly typeset in posts, which makes them easier to read.
      $endgroup$
      – hardmath
      Jan 24 at 16:40




      1




      1




      $begingroup$
      @PaulSinclair In the solution , $n=2^j$ , so the pattern does indeed follow , as $$frac{1}{2^2^{j+1}-1}=frac{1}{(2^2^j+1)(2^2^j-1)}$$ . While the solution should have mentioned this , I hope this clears it up .
      $endgroup$
      – Sinπ
      Jan 24 at 19:05






      $begingroup$
      @PaulSinclair In the solution , $n=2^j$ , so the pattern does indeed follow , as $$frac{1}{2^2^{j+1}-1}=frac{1}{(2^2^j+1)(2^2^j-1)}$$ . While the solution should have mentioned this , I hope this clears it up .
      $endgroup$
      – Sinπ
      Jan 24 at 19:05














      $begingroup$
      @Rahuboy For future reference you can't do double exponentials without braces, e.g. $2^{2^{j+1}}$ is 2^{2^{j+1}}, not 2^2^{j+1}. That's why your comment malfunctioned.
      $endgroup$
      – J.G.
      Jan 24 at 22:29




      $begingroup$
      @Rahuboy For future reference you can't do double exponentials without braces, e.g. $2^{2^{j+1}}$ is 2^{2^{j+1}}, not 2^2^{j+1}. That's why your comment malfunctioned.
      $endgroup$
      – J.G.
      Jan 24 at 22:29












      $begingroup$
      @J.G. Oh I see.... thanks for the help !
      $endgroup$
      – Sinπ
      Jan 25 at 0:02




      $begingroup$
      @J.G. Oh I see.... thanks for the help !
      $endgroup$
      – Sinπ
      Jan 25 at 0:02












      $begingroup$
      That is embarrassing. My apologies for failing to realize this.
      $endgroup$
      – Paul Sinclair
      Jan 25 at 0:14




      $begingroup$
      That is embarrassing. My apologies for failing to realize this.
      $endgroup$
      – Paul Sinclair
      Jan 25 at 0:14











      7












      $begingroup$

      You may prove by induction that $$sum_{r=0}^nfrac{2^{2^r}}{2^{2^{r+1}}-1}=1-frac{1}{2^{2^{n+1}}-1}.$$(If yoiu didn't spot this conjecture at first, you will after calculating the first few partial sums.) Indeed the claim is correct if $n=0$, and if it holds for $n=k$ then $$sum_{r=0}^{k+1}frac{2^{2^r}}{2^{2^{r+1}}-1}=1-frac{1}{2^{2^{k+1}}-1}+frac{2^{2^{k+1}}}{2^{2^{k+2}}-1}=1-frac{1}{2^{2^{k+2}}-1}$$as required, where the final calculation is the $a=2^{2^{k+1}}$ special case of $$1-frac{1}{a-1}+frac{a}{a^2-1}=1-frac{1}{a^2-1}.$$You may wish to rewrite this argument as the computation of a telescoping series.






      share|cite|improve this answer









      $endgroup$


















        7












        $begingroup$

        You may prove by induction that $$sum_{r=0}^nfrac{2^{2^r}}{2^{2^{r+1}}-1}=1-frac{1}{2^{2^{n+1}}-1}.$$(If yoiu didn't spot this conjecture at first, you will after calculating the first few partial sums.) Indeed the claim is correct if $n=0$, and if it holds for $n=k$ then $$sum_{r=0}^{k+1}frac{2^{2^r}}{2^{2^{r+1}}-1}=1-frac{1}{2^{2^{k+1}}-1}+frac{2^{2^{k+1}}}{2^{2^{k+2}}-1}=1-frac{1}{2^{2^{k+2}}-1}$$as required, where the final calculation is the $a=2^{2^{k+1}}$ special case of $$1-frac{1}{a-1}+frac{a}{a^2-1}=1-frac{1}{a^2-1}.$$You may wish to rewrite this argument as the computation of a telescoping series.






        share|cite|improve this answer









        $endgroup$
















          7












          7








          7





          $begingroup$

          You may prove by induction that $$sum_{r=0}^nfrac{2^{2^r}}{2^{2^{r+1}}-1}=1-frac{1}{2^{2^{n+1}}-1}.$$(If yoiu didn't spot this conjecture at first, you will after calculating the first few partial sums.) Indeed the claim is correct if $n=0$, and if it holds for $n=k$ then $$sum_{r=0}^{k+1}frac{2^{2^r}}{2^{2^{r+1}}-1}=1-frac{1}{2^{2^{k+1}}-1}+frac{2^{2^{k+1}}}{2^{2^{k+2}}-1}=1-frac{1}{2^{2^{k+2}}-1}$$as required, where the final calculation is the $a=2^{2^{k+1}}$ special case of $$1-frac{1}{a-1}+frac{a}{a^2-1}=1-frac{1}{a^2-1}.$$You may wish to rewrite this argument as the computation of a telescoping series.






          share|cite|improve this answer









          $endgroup$



          You may prove by induction that $$sum_{r=0}^nfrac{2^{2^r}}{2^{2^{r+1}}-1}=1-frac{1}{2^{2^{n+1}}-1}.$$(If yoiu didn't spot this conjecture at first, you will after calculating the first few partial sums.) Indeed the claim is correct if $n=0$, and if it holds for $n=k$ then $$sum_{r=0}^{k+1}frac{2^{2^r}}{2^{2^{r+1}}-1}=1-frac{1}{2^{2^{k+1}}-1}+frac{2^{2^{k+1}}}{2^{2^{k+2}}-1}=1-frac{1}{2^{2^{k+2}}-1}$$as required, where the final calculation is the $a=2^{2^{k+1}}$ special case of $$1-frac{1}{a-1}+frac{a}{a^2-1}=1-frac{1}{a^2-1}.$$You may wish to rewrite this argument as the computation of a telescoping series.







          share|cite|improve this answer












          share|cite|improve this answer



          share|cite|improve this answer










          answered Jan 24 at 13:13









          J.G.J.G.

          30.3k23148




          30.3k23148























              2












              $begingroup$

              Note first that
              $$
              frac{2^n}{2^{2n}-1}=2^{-n}frac{1}{1-2^{-2n}}=sum_{k:text{odd},kinBbb N} 2^{-nk}.
              $$
              If we sum over $n=2^j$, $jge 0$, we have
              $$
              S=sum_{j=0}^infty frac{2^{2^j}}{2^{2^{j+1}}-1}=sum_{j=0}^inftysum_{k:text{odd},kinBbb N} 2^{-2^jcdot k}=sum_{l=1}^infty 2^{-l}=1
              $$
              since every $lge 1$ has a unique representation $l=2^jcdot k$ for some $jge 0$ and odd $kin Bbb N$.






              share|cite|improve this answer











              $endgroup$


















                2












                $begingroup$

                Note first that
                $$
                frac{2^n}{2^{2n}-1}=2^{-n}frac{1}{1-2^{-2n}}=sum_{k:text{odd},kinBbb N} 2^{-nk}.
                $$
                If we sum over $n=2^j$, $jge 0$, we have
                $$
                S=sum_{j=0}^infty frac{2^{2^j}}{2^{2^{j+1}}-1}=sum_{j=0}^inftysum_{k:text{odd},kinBbb N} 2^{-2^jcdot k}=sum_{l=1}^infty 2^{-l}=1
                $$
                since every $lge 1$ has a unique representation $l=2^jcdot k$ for some $jge 0$ and odd $kin Bbb N$.






                share|cite|improve this answer











                $endgroup$
















                  2












                  2








                  2





                  $begingroup$

                  Note first that
                  $$
                  frac{2^n}{2^{2n}-1}=2^{-n}frac{1}{1-2^{-2n}}=sum_{k:text{odd},kinBbb N} 2^{-nk}.
                  $$
                  If we sum over $n=2^j$, $jge 0$, we have
                  $$
                  S=sum_{j=0}^infty frac{2^{2^j}}{2^{2^{j+1}}-1}=sum_{j=0}^inftysum_{k:text{odd},kinBbb N} 2^{-2^jcdot k}=sum_{l=1}^infty 2^{-l}=1
                  $$
                  since every $lge 1$ has a unique representation $l=2^jcdot k$ for some $jge 0$ and odd $kin Bbb N$.






                  share|cite|improve this answer











                  $endgroup$



                  Note first that
                  $$
                  frac{2^n}{2^{2n}-1}=2^{-n}frac{1}{1-2^{-2n}}=sum_{k:text{odd},kinBbb N} 2^{-nk}.
                  $$
                  If we sum over $n=2^j$, $jge 0$, we have
                  $$
                  S=sum_{j=0}^infty frac{2^{2^j}}{2^{2^{j+1}}-1}=sum_{j=0}^inftysum_{k:text{odd},kinBbb N} 2^{-2^jcdot k}=sum_{l=1}^infty 2^{-l}=1
                  $$
                  since every $lge 1$ has a unique representation $l=2^jcdot k$ for some $jge 0$ and odd $kin Bbb N$.







                  share|cite|improve this answer














                  share|cite|improve this answer



                  share|cite|improve this answer








                  edited Jan 24 at 13:19

























                  answered Jan 24 at 13:11









                  SongSong

                  18k21449




                  18k21449






























                      draft saved

                      draft discarded




















































                      Thanks for contributing an answer to Mathematics Stack Exchange!


                      • Please be sure to answer the question. Provide details and share your research!

                      But avoid



                      • Asking for help, clarification, or responding to other answers.

                      • Making statements based on opinion; back them up with references or personal experience.


                      Use MathJax to format equations. MathJax reference.


                      To learn more, see our tips on writing great answers.




                      draft saved


                      draft discarded














                      StackExchange.ready(
                      function () {
                      StackExchange.openid.initPostLogin('.new-post-login', 'https%3a%2f%2fmath.stackexchange.com%2fquestions%2f3085853%2finfinite-series-problem%23new-answer', 'question_page');
                      }
                      );

                      Post as a guest















                      Required, but never shown





















































                      Required, but never shown














                      Required, but never shown












                      Required, but never shown







                      Required, but never shown

































                      Required, but never shown














                      Required, but never shown












                      Required, but never shown







                      Required, but never shown







                      Popular posts from this blog

                      Can a sorcerer learn a 5th-level spell early by creating spell slots using the Font of Magic feature?

                      Does disintegrating a polymorphed enemy still kill it after the 2018 errata?

                      A Topological Invariant for $pi_3(U(n))$